Buy BOOKS at Discounted Price

Playing With Numbers

Class 8th Mathematics RS Aggarwal Solution
Exercise 5a
  1. The units digit of a two-digit number is 3 and seven times the sum of the digits…
  2. In a two-digit number, the digit at the unit place is double the digit in the…
  3. A two-digit number is 3 more than 4 times the sum of its digits. If 18 is added…
  4. The sum of the digits of a two-digit number is 15. The number obtained by…
  5. The difference between a 2-digit number and the number obtained by interchanging…
  6. In a 3-digit number, the tens digit is thrice the units digit and the hundreds…
Exercise 5b
  1. Test the divisibility of each of the following numbers by 2: (i) 94 (ii) 570…
  2. Test the divisibility of each of the following numbers by 5: (i) 95 (ii) 470…
  3. Test the divisibility of each of the following numbers by 10: (i) 205 (ii) 90…
  4. Test the divisibility of each of the following numbers by 3: (i) 83 (ii) 378…
  5. Test the divisibility of each of the following numbers by 9: (i) 327 (ii) 7524…
  6. Test the divisibility of each of the following numbers by 4: (i) 134 (ii) 618…
  7. Test the divisibility of each of the following numbers by 8: (i) 6132 (ii) 7304…
  8. Test the divisibility of each of the following numbers by 11: (i) 22222 (ii)…
  9. Test the divisibility of each of the following numbers by 7: (i) 693 (ii) 7896…
  10. Find all possible values of x for which the number 7x3 is divisible by 3. Also,…
  11. Find all possible values of y for which the number 53y1 is divisible by 3.…
  12. Find the value of x for which the number x806 is divisible by 9. Also, find the…
  13. Find the value of z for which the number 471z8 is divisible by 9. Also, find…
  14. Give five examples of numbers, each one of which is divisible by 3 but not…
  15. Give five examples of numbers, each one of which is divisible by 4 but not…
Exercise 5c
  1. Replace A, B, C by suitable numerals.
  2. r 4cb6 + 369a 8173 Replace A, B, C by suitable numerals.
  3. r a + a + a b Replace A, B by suitable numerals.
  4. Replace A, B by suitable numerals.
  5. Replace A, B, C by suitable numerals.
  6. r ab x 3 cab Replace A, B, C by suitable numbers:
  7. r ab x b Replace A, B, C by suitable numbers:
  8. Replace A, B, C by suitable numerals.
  9. Find two numbers whose product is a 1-digit number and the sum is a 2-digit…
  10. Find three whole numbers whose product and sum are equal.
  11. Complete the magic square given below, so that the sum of the numbers in each…
  12. Fill in the numbers from 1 to 6 without repetition, so that each side of the…
  13. Fibonacci numbers Take 10 numbers as shown below: a, b, (a + b), (a + 2b), (2a…
  14. Complete the magic square: 14 0 8 6 11 4 7 2 1 12
Exercise 5d
  1. If 5x6 is exactly divisible by 3, then the least value of x isA. 0 B. 1 C. 2 D.…
  2. If 64y8 is exactly divisible by 3, then the least value of y isA. 0 B. 1 C. 2 D.…
  3. If 7x8 is exactly divisible by 9, then the least value of x isA. 0 B. 2 C. 3 D.…
  4. If 37y4 is exactly divisible by 9, then the least value of y isA. 2 B. 3 C. 1 D.…
  5. If 4xy7 is exactly divisible by 3, then the least value of (x + y) isA. 1 B. 4…
  6. If x7y5 is exactly divisible by 3, then the least value of (x + y) isA. 6 B. 0…
  7. If x4y5z is exactly divisible by 9, then the least value of (x + y + z) isA. 3…
  8. If 1A2B5 is exactly divisible by 9, then the least value of (A + B) isA. 0 B. 1…
  9. If the 4-digit number x27y is exactly divisible by 9, then the least value of (x…
Cce Test Paper-5
  1. Find all possible values of x for which the 4-digit number 320x is divisible by 3.…
  2. Find all possible values of y for which the 4-digit number 64y3 is divisible by 9.…
  3. The sum of the digits of a 2-digit number is 6. The number obtained by interchanging…
  4. Which of the following numbers are divisible by 9? (i) 524618 (ii) 7345845 (iii)…
  5. Replace A, B, C by suitable numerals:
  6. c 7 long / 6ab (8c -56/6 b -63/x Replace A, B, C by suitable numerals:…
  7. Find the values of A, B, C when
  8. If 7 8 is exactly divisible by 3, then the least value of x isA. 3 B. 0 C. 6 D. 9…
  9. If 6x5 is exactly divisible by 9, then the least value of x isA. 1 B. 4 C. 7 D. 0…
  10. If x48y is exactly divisible by 9, then the least value of (x + y) isA. 4 B. 0 C. 6 D.…
  11. If 486*7 is divisible by 9, then the least value of * isA. 0 B. 1 C. 3 D. 2…

Exercise 5a
Question 1.

The units digit of a two-digit number is 3 and seven times the sum of the digits is the number itself. Find the number.


Answer:

It is given that the units place digit is 3.


So, let tens place digit be y.


∴ Our number = (10y + 3) …(1)


Our given condition is that seven times the sum of the digits is the number itself.


∴ By given condition,


7(y + 3) = (10 y + 3)
7 y + 21 = 10 y + 3
∴ 10 y - 7y = 21 - 3
∴ 3 y = 18
∴ y = 6


Substituting the value of y in equation1,


Number = 10 × 6 + 3 = 63


Hence, required number is 63.



Question 2.

In a two-digit number, the digit at the unit place is double the digit in the tens place. The number exceeds the sum of its digits by 18. Find the number.


Answer:

Our first given condition is that the digit at the unit place is double the digit in the tens place.


∴ Let the tens digit be y.
The digit in the units place is 2y.
Number = 10y + 2y = 12y


Now the second condition is that the number exceeds the sum of its digits by 18.


∴ By given condition,


(y + 2y) + 18 = (10y + 2y)
∴ 3y + 18 = 12y
12y - 3y = 18
9y =18


∴ y = 2


Hence, the digit in the tens place is 2.


So, digit in the units place is 4.


∴ Our number is 24.



Question 3.

A two-digit number is 3 more than 4 times the sum of its digits. If 18 is added to the number, its digits are reversed. Find the number.


Answer:

Let tens place digit be y and the units place be x.


∴ Our number is (10y + x)


Our given first condition is that our number is 3 more than 4 times the sum of its digits.


∴ By given condition,


4(y + x) + 3 = (10 y + x)
4y + 4x + 3 = 10y + x
6y - 3x = 3
3(2y - x) = 3
2y - x =1 …(1)


Our given second condition is that if 18 is added to the number, its digits are reversed.


The reversed number is (10x + y)


∴ By given condition,


(10y + x) + 18 = 10x + y
10y - y + x -10x = -18
9y - 9x = -18
9(y - x) = -18
y - x = -2
y = x - 2 [2]

Putting this value of 'y' in eq (1), we have
2(x - 2) - x = 1
2x - 4 - x = 1
x = 5

From [2], we have
y = 5 - 2 = 3

Hence,

y = 3 and x = 5

∴ Our number = (10 × 3 + 5) = 35

Hence, our number is 35.


Question 4.

The sum of the digits of a two-digit number is 15. The number obtained by interchanging its digits exceeds the given number by 9. Find the original number.


Answer:

Let tens place digit be y and the units place be x.


∴ Our number is (10y + x)


Our given first condition is that sum of the digits of a two-digit number is 15.


∴ By given condition,


y + x = 15 …(1)


Our given second condition is that the number obtained by interchanging its digits exceeds the given number by 9.


∴ By given condition,


10y + x + 9 = 10 x + y
∴ 10y - y + x - 10x = -9
9y - 9x = -9​
y - x = -1 …(2)


Solving 1 and 2 simultaneously, we get,


∴ y = 7 and x = 8


∴ Our number = (10 × 7 + 8) = 78


Hence, our number is 78.


(Answer given is 48 but correct answer is 78)



Question 5.

The difference between a 2-digit number and the number obtained by interchanging its digits is 63. What is the difference between the digits of the number?


Answer:

Let tens place digit be y and the units place be x.


∴ Our number is (10y + x)


Our given first condition is that the difference between a 2-digit number and the number obtained by interchanging its digits is 63.


∴ By given condition,


(10y + x) - (10x + y) = 63
∴ 10y - y + x - 10 x = 63
9y – 9x = 63
9(y - x) = 63
y - x = 7


Hence, the difference between the digits of the number is 7.



Question 6.

In a 3-digit number, the tens digit is thrice the units digit and the hundreds digit is four times the units digit. Also, the sum of its digits is 16. Find the number.


Answer:

Let the units place digit be x.


Our given condition is that the tens digit is thrice the units digit and the hundreds digit is four times the units digit and sum of digits is 16.


∴ By given condition,


4x + 3x + x = 16
8x = 16
x =2


∴ The number is 862.




Exercise 5b
Question 1.

Test the divisibility of each of the following numbers by 2:

(i) 94

(ii) 570

(iii) 285

(iv) 2398

(v) 79532

(vi) 13576

(vii) 46821

(viii) 84663

(ix) 66669


Answer:

(i) 94


We know that if units place digit is divisible by 2 then our given number is divisible by 2.


Here units place is 4, which is divisible by 2.


Hence, 94 is divisible by 2.


(ii) 570


We know that if units place digit is divisible by 2 then our given number is divisible by 2.


Here units place is 0, which is divisible by 2.


Hence, 570 is divisible by 2.


(iii) 285


We know that if units place digit is divisible by 2 then our given number is divisible by 2.


Here units place is 5, which is not divisible by 2.


Hence, 570 is not divisible by 2.


(iv) 2398


We know that if units place digit is divisible by 2 then our given number is divisible by 2.


Here units place is 8, which is divisible by 2.


Hence, 2398 is divisible by 2.


(v) 79532


We know that if units place digit is divisible by 2 then our given number is divisible by 2.


Here units place is 8, which is divisible by 2.


Hence, 79532 is divisible by 2.


(vi) 13576


We know that if units place digit is divisible by 2 then our given number is divisible by 2.


Here units place is 6, which is divisible by 2.


Hence, 13576 is divisible by 2.


(vii) 46821


We know that if units place digit is divisible by 2 then our given number is divisible by 2.


Here units place is 1, which is not divisible by 2.


Hence, 46821 is not divisible by 2.


(viii) 84663


We know that if units place digit is divisible by 2 then our given number is divisible by 2.


Here units place is 3, which is not divisible by 2.


Hence, 84663 is not divisible by 2.


(ix) 66669


We know that if units place digit is divisible by 2 then our given number is divisible by 2.


Here units place is 9, which is not divisible by 2.


Hence, 66669 is not divisible by 2.



Question 2.

Test the divisibility of each of the following numbers by 5:

(i) 95

(ii) 470

(iii) 1056

(iv) 2735

(v) 55053

(vi) 35790

(vii) 98765

(viii) 42658

(ix) 77990


Answer:

(i) 95


We know that if units place is divisible by 5, ie. units place is having value either of 0 or 5, then our number is divisible by 5.


Here units place is 5, which is divisible by 5.


Hence, 95 is divisible by 5.


(ii) 470


We know that if units place is divisible by 5, ie. units place is having value either of 0 or 5, then our number is divisible by 5.


Here units place is 0, which is divisible by 5.


Hence, 470 is divisible by 5.


(iii) 1056


We know that if units place is divisible by 5, ie. units place is having value either of 0 or 5, then our number is divisible by 5.


Here units place is 6, which is not divisible by 5.


Hence, 1056 is not divisible by 5.


(iv) 2735


We know that if units place is divisible by 5, ie. units place is having value either of 0 or 5, then our number is divisible by 5.


Here units place is 5, which is divisible by 5.


Hence, 2735 is divisible by 5.


(v) 55053


We know that if units place is divisible by 5, ie. units place is having value either of 0 or 5, then our number is divisible by 5.


Here units place is 3, which is not divisible by 5.


Hence, 55053 is not divisible by 5.


(vi) 35790


We know that if units place is divisible by 5, ie. units place is having value either of 0 or 5, then our number is divisible by 5.


Here units place is 0, which is divisible by 5.


Hence, 35790 is divisible by 5.


(vii) 98765


We know that if units place is divisible by 5, ie. units place is having value either of 0 or 5, then our number is divisible by 5.


Here units place is 5, which is divisible by 5.


Hence, 98765 is divisible by 5.


(viii) 42658


We know that if units place is divisible by 5, ie. units place is having value either of 0 or 5, then our number is divisible by 5.


Here units place is 8, which is not divisible by 5.


Hence, 42658 is not divisible by 5.


(ix) 77990


We know that if units place is divisible by 5, ie. units place is having value either of 0 or 5, then our number is divisible by 5.


Here units place is 0, which is divisible by 5.


Hence, 77990 is divisible by 5.



Question 3.

Test the divisibility of each of the following numbers by 10:

(i) 205

(ii) 90

(iii) 1174

(iv) 57930

(v) 60005


Answer:

(i) 205


We know that if units place is divisible by 10, ie. units place is having value 0, then our number is divisible by 10.


Here units place is 5, which is not divisible by 10.


Hence, 205 is not divisible by 10.


(ii) 90


We know that if units place is divisible by 10, ie. units place is having value 0, then our number is divisible by 10.


Here units place is 0, which is divisible by 10.


Hence, 90 is divisible by 10.


(iii) 1174


We know that if units place is divisible by 10, ie. units place is having value 0, then our number is divisible by 10.


Here units place is 4, which is not divisible by 10.


Hence, 1174 is not divisible by 10.


(iv) 57930


We know that if units place is divisible by 10, ie. units place is having value 0, then our number is divisible by 10.


Here units place is 0, which is divisible by 10.


Hence, 57930 is divisible by 10.


(v) 60005


We know that if units place is divisible by 10, ie. units place is having value 0, then our number is divisible by 10.


Here units place is 5, which is not divisible by 10.


Hence, 60005 is not divisible by 10.



Question 4.

Test the divisibility of each of the following numbers by 3:

(i) 83

(ii) 378

(iii) 474

(iv) 1693

(v) 60005

(vi) 67035

(vii) 591282

(viii) 903164

(ix) 100002


Answer:

(i) 83


We know that if sum of digits of a number is divisible by 3, then the number is divisible by 3.


Here, sum of digits is 11, which is not divisible by 3.


Hence, 83 is not divisible by 3.


(ii) 378


We know that if sum of digits of a number is divisible by 3, then the number is divisible by 3.


Here, sum of digits is 18, which is divisible by 3.


Hence, 378 is divisible by 3.


(iii) 474


We know that if sum of digits of a number is divisible by 3, then the number is divisible by 3.


Here, sum of digits is 15, which is divisible by 3.


Hence, 474 is divisible by 3.


(iv) 1693


We know that if sum of digits of a number is divisible by 3, then the number is divisible by 3.


Here, sum of digits is 19, which is not divisible by 3.


Hence, 1693 is not divisible by 3.


(v) 60005


We know that if sum of digits of a number is divisible by 3, then the number is divisible by 3.


Here, sum of digits is 11, which is not divisible by 3.


Hence, 60005 is not divisible by 3.


(vi) 67035


We know that if sum of digits of a number is divisible by 3, then the number is divisible by 3.


Here, sum of digits is 21, which is divisible by 3.


Hence, 67035 is divisible by 3.


(vii) 591282


We know that if sum of digits of a number is divisible by 3, then the number is divisible by 3.


Here, sum of digits is 27, which is divisible by 3.


Hence, 591282 is divisible by 3.


(viii) 903164


We know that if sum of digits of a number is divisible by 3, then the number is divisible by 3.


Here, sum of digits is 23, which is not divisible by 3.


Hence, 903164 is not divisible by 3.


(ix) 100002


We know that if sum of digits of a number is divisible by 3, then the number is divisible by 3.


Here, sum of digits is 3, which is divisible by 3.


Hence, 100002 is divisible by 3.



Question 5.

Test the divisibility of each of the following numbers by 9:

(i) 327

(ii) 7524

(iii) 32022

(iv) 64302

(v) 89361

(vi) 14799

(vii) 66888

(viii) 30006

(ix) 33333


Answer:

(i) 327


We know that if sum of digits of a number is divisible by 9, then the number is divisible by 9.


Here, sum of digits is 12, which is not divisible by 9.


Hence, 327 is not divisible by 9.


(ii) 7524


We know that if sum of digits of a number is divisible by 9, then the number is divisible by 9.


Here, sum of digits is 18, which is divisible by 9.


Hence, 7524 is divisible by 9.


(iii) 32022


We know that if sum of digits of a number is divisible by 9, then the number is divisible by 9.


Here, sum of digits is 9, which is divisible by 9.


Hence, 32022 is divisible by 9.


(iv) 64302


We know that if sum of digits of a number is divisible by 9, then the number is divisible by 9.


Here, sum of digits is 15, which is not divisible by 9.


Hence, 64302 is not divisible by 9.


(v) 89361


We know that if sum of digits of a number is divisible by 9, then the number is divisible by 9.


Here, sum of digits is 27, which is divisible by 9.


Hence, 89361 is divisible by 9.


(vi) 14799


We know that if sum of digits of a number is divisible by 9, then the number is divisible by 9.


Here, sum of digits is 30, which is not divisible by 9.


Hence, 14799 is not divisible by 9.


(vii) 66888


We know that if sum of digits of a number is divisible by 9, then the number is divisible by 9.


Here, sum of digits is 36, which is divisible by 9.


Hence, 66888 is divisible by 9.


(viii) 30006


We know that if sum of digits of a number is divisible by 9, then the number is divisible by 9.


Here, sum of digits is 9, which is divisible by 9.


Hence, 30006 is divisible by 9.


(ix) 33333


We know that if sum of digits of a number is divisible by 9, then the number is divisible by 9.


Here, sum of digits is 15, which is not divisible by 9.


Hence, 33333 is not divisible by 9.



Question 6.

Test the divisibility of each of the following numbers by 4:

(i) 134

(ii) 618

(iii) 3928

(iv) 50176

(v) 39392

(vi) 56794

(vii) 86102

(viii) 66666

(ix) 99918

(x) 77736


Answer:

We know that if number formed by last two digits of a given number is divisible by 4, then entire given number is divisible by 4.


Here, number formed by last two digits is 34, which is not divisible by 4.


Hence, 134 is not divisible by 4.


(ii) 618


We know that if number formed by last two digits of a given number is divisible by 4, then entire given number is divisible by 4.


Here, number formed by last two digits is 18, which is not divisible by 4.


Hence, 618 is not divisible by 4.


(iii) 3928


We know that if number formed by last two digits of a given number is divisible by 4, then entire given number is divisible by 4.


Here, number formed by last two digits is 28, which is divisible by 4.


Hence, 3928 is divisible by 4.


(iv) 50176


We know that if number formed by last two digits of a given number is divisible by 4, then entire given number is divisible by 4.


Here, number formed by last two digits is 76, which is divisible by 4.


Hence, 50176 is divisible by 4.


(v) 39392


We know that if number formed by last two digits of a given number is divisible by 4, then entire given number is divisible by 4.


Here, number formed by last two digits is 92, which is divisible by 4.


Hence, 39392 is divisible by 4.


(vi) 56794


We know that if number formed by last two digits of a given number is divisible by 4, then entire given number is divisible by 4.


Here, number formed by last two digits is 94, which is not divisible by 4.


Hence, 56794 is not divisible by 4.


(vii) 86102


We know that if number formed by last two digits of a given number is divisible by 4, then entire given number is divisible by 4.


Here, number formed by last two digits is 02, which is not divisible by 4.


Hence, 86102 is not divisible by 4.


(viii) 66666


We know that if number formed by last two digits of a given number is divisible by 4, then entire given number is divisible by 4.


Here, number formed by last two digits is 66, which is not divisible by 4.


Hence, 66666 is not divisible by 4.


(ix) 99918


We know that if number formed by last two digits of a given number is divisible by 4, then entire given number is divisible by 4.


Here, number formed by last two digits is 18, which is not divisible by 4.


Hence, 99918 is not divisible by 4.


(x) 77736


We know that if number formed by last two digits of a given number is divisible by 4, then entire given number is divisible by 4.


Here, number formed by last two digits is 36, which is divisible by 4.


Hence, 77736 is divisible by 4.



Question 7.

Test the divisibility of each of the following numbers by 8:

(i) 6132

(ii) 7304

(iii) 59312

(iv) 66664

(v) 44444

(vi) 154360

(vii) 998818

(viii) 265472

(ix) 7350162


Answer:

(i) 6132


We know that if number formed by last three digits of a given number is divisible by 8, then entire given number is divisible by 8.


Here, number formed by last three digits is 132, which is not divisible by 8.


Hence, 6132 is not divisible by 8.


(ii) 7304


We know that if number formed by last three digits of a given number is divisible by 8, then entire given number is divisible by 8.


Here, number formed by last three digits is 304, which is divisible by 8.


Hence, 7304 is divisible by 8.


(iii) 59312


We know that if number formed by last three digits of a given number is divisible by 8, then entire given number is divisible by 8.


Here, number formed by last three digits is 312, which is divisible by 8.


Hence, 59312 is divisible by 8.


(iv) 66664


We know that if number formed by last three digits of a given number is divisible by 8, then entire given number is divisible by 8.


Here, number formed by last three digits is 664, which is divisible by 8.


Hence, 66664 is divisible by 8.


(v) 44444


We know that if number formed by last three digits of a given number is divisible by 8, then entire given number is divisible by 8.


Here, number formed by last three digits is 444, which is not divisible by 8.


Hence, 44444 is not divisible by 8.


(vi) 154360


We know that if number formed by last three digits of a given number is divisible by 8, then entire given number is divisible by 8.


Here, number formed by last three digits is 360, which is divisible by 8.


Hence, 154360 is divisible by 8.


(vii) 998818


We know that if number formed by last three digits of a given number is divisible by 8, then entire given number is divisible by 8.


Here, number formed by last three digits is 818, which is not divisible by 8.


Hence, 998818 is not divisible by 8.


(viii) 265472


We know that if number formed by last three digits of a given number is divisible by 8, then entire given number is divisible by 8.


Here, number formed by last three digits is 472, which is divisible by 8.


Hence, 265472 is divisible by 8.


(ix) 7350162


We know that if number formed by last three digits of a given number is divisible by 8, then entire given number is divisible by 8.


Here, number formed by last three digits is 162, which is not divisible by 8.


Hence, 7350162 is not divisible by 8.



Question 8.

Test the divisibility of each of the following numbers by 11:

(i) 22222

(ii) 444444

(iii) 379654

(iv) 1057982

(v) 6543207

(vi) 818532

(vii) 900163

(viii) 7531622


Answer:

(i) 22222


We know that if the difference of the sum of alternative digits of a number, i.e. digits which are in odd places together and digits in even places together, is divisible by 11 then that number is divisible by 11.


Here, sum of digits in odd places = 6 and sum of digits in even places = 4


∴ The difference of the sum of alternative digits of a number is 2, which is not divisible by 11.


Hence, 22222 is not divisible by 11.


(ii) 444444


We know that if the difference of the sum of alternative digits of a number, i.e. digits which are in odd places together and digits in even places together, is divisible by 11 then that number is divisible by 11.


Here, sum of digits in odd places = 12 and sum of digits in even places = 12


∴ The difference of the sum of alternative digits of a number is 0, which is divisible by 11.


Hence, 444444 is divisible by 11.


(iii) 379654


We know that if the difference of the sum of alternative digits of a number, i.e. digits which are in odd places together and digits in even places together, is divisible by 11 then that number is divisible by 11.


Here, sum of digits in odd places = 17 and sum of digits in even places = 17


∴ The difference of the sum of alternative digits of a number is 0, which is divisible by 11.


Hence, 379654 is divisible by 11.


(iv) 1057982


We know that if the difference of the sum of alternative digits of a number, i.e. digits which are in odd places together and digits in even places together, is divisible by 11 then that number is divisible by 11.


Here, sum of digits in odd places = 17 and sum of digits in even places = 15


∴ The difference of the sum of alternative digits of a number is 2, which is not divisible by 11.


Hence, 1057982 is not divisible by 11.


(v) 6543207


We know that if the difference of the sum of alternative digits of a number, i.e. digits which are in odd places together and digits in even places together, is divisible by 11 then that number is divisible by 11.


Here, sum of digits in odd places = 19 and sum of digits in even places = 8


∴ The difference of the sum of alternative digits of a number is 11, which is divisible by 11.


Hence, 6543207 is divisible by 11.


(vi) 818532


We know that if the difference of the sum of alternative digits of a number, i.e. digits which are in odd places together and digits in even places together, is divisible by 11 then that number is divisible by 11.


Here, sum of digits in odd places = 8 and sum of digits in even places = 19


∴ The difference of the sum of alternative digits of a number is 11, which is divisible by 11.


Hence, 818532 is divisible by 11.


(vii) 900163


We know that if the difference of the sum of alternative digits of a number, i.e. digits which are in odd places together and digits in even places together, is divisible by 11 then that number is divisible by 11.


Here, sum of digits in odd places = 4 and sum of digits in even places = 15


∴ The difference of the sum of alternative digits of a number is 11, which is divisible by 11.


Hence, 900163 is divisible by 11.


(viii) 7531622


We know that if the difference of the sum of alternative digits of a number, i.e. digits which are in odd places together and digits in even places together, is divisible by 11 then that number is divisible by 11.


Here, sum of digits in odd places = 18 and sum of digits in even places = 8


∴ The difference of the sum of alternative digits of a number is 10, which is not divisible by 11.


Hence, 7531622 is not divisible by 11.



Question 9.

Test the divisibility of each of the following numbers by 7:

(i) 693

(ii) 7896

(iii) 3467

(iv) 12873

(v) 65436

(vi) 54636

(vii) 98175

(viii) 88777


Answer:

(i) 693


We know that if number formed by removing last digit is subtracted by double of removed digit, is divisible by 7, then our given number is divisible by 7.


Here last digit is 3.


∴ Number to be tested = 69 – 2 × 3 = 63, which is divisible by 7.


Hence, 693 is divisible by 7.


(ii) 7896


We know that if number formed by removing last digit is subtracted by double of removed digit, is divisible by 7, then our given number is divisible by 7.


Here last digit is 6.


∴ Number to be tested = 789 – 2 × 6 = 777, which is divisible by 7.


Hence, 7896 is divisible by 7.


(iii) 3467


We know that if number formed by removing last digit is subtracted by double of removed digit, is divisible by 7, then our given number is divisible by 7.


Here last digit is 7.


∴ Number to be tested = 346 – 2 × 7 = 332.


Again repeating the process, taking last digit 2 for number 332,


Number to be tested = 33 – 2 × 4 = 19, which is not divisible by 7.


Hence, 3467 is not divisible by 7.


(iv) 12873


We know that if number formed by removing last digit is subtracted by double of removed digit, is divisible by 7, then our given number is divisible by 7.


Here last digit is 3.


∴ Number to be tested = 1287 – 2 × 3 = 1281.


Again repeating the process, taking last digit 1 for number 1281,


Number to be tested = 128 – 2 × 1 = 126, which is divisible by 7.


Hence, 12873 is divisible by 7.


(v) 65436


We know that if number formed by removing last digit is subtracted by double of removed digit, is divisible by 7, then our given number is divisible by 7.


Here last digit is 6.


∴ Number to be tested = 6543 – 2 × 6 = 6531.


Again repeating the process, taking last digit 1 for number 6531,


Number to be tested = 653 – 2 × 1 = 651


Again repeating the process, taking last digit 1 for number 651,


Number to be tested = 65 – 2 × 1 = 63, which is divisible by 7.


Hence, 65436 is divisible by 7.


(vi) 54636


We know that if number formed by removing last digit is subtracted by double of removed digit, is divisible by 7, then our given number is divisible by 7.


Here last digit is 6.


∴ Number to be tested = 5463 – 2 × 6 = 5451


Again repeating the process, taking last digit 1 for number 5451,


Number to be tested = 545 – 2 × 1 = 543


Again repeating the process, taking last digit 3 for number 543,


Number to be tested = 54 – 2 × 3 = 48, which is not divisible by 7.


Hence, 54636 is not divisible by 7.


(vii) 98175


We know that if number formed by removing last digit is subtracted by double of removed digit, is divisible by 7, then our given number is divisible by 7.


Here last digit is 5.


∴ Number to be tested = 9817 – 2 × 5 = 9807.


Again repeating the process, taking last digit 7 for number 9807,


Number to be tested = 980 – 2 × 7 = 966


Again repeating the process, taking last digit 6 for number 966,


Number to be tested = 96 – 2 × 6 = 84, which is divisible by 7.


Hence, 98175 is divisible by 7.


(viii) 88777


We know that if number formed by removing last digit is subtracted by double of removed digit, is divisible by 7, then our given number is divisible by 7.


Here last digit is 7.


∴ Number to be tested = 8877 – 2 × 7 = 8863.


Again repeating the process, taking last digit 3 for number 8863,


Number to be tested = 886 – 2 × 3 = 880


Here, last digit is 0. So, last digit will not be considered.


Hence, our number testing number will be 88, which is not divisible by 7.


Hence, 88777 is not divisible by 7.



Question 10.

Find all possible values of x for which the number 7x3 is divisible by 3. Also, find each such number.


Answer:

Here, our given number is 7x3, which is divisible by 3.


We know that if number is divisible by 3 only when sum of digits of given number is divisible by 3.


Hence, if 7x3 is divisible by 3, then,


7 + x + 3 = 0, 3, 6, 9,12,…


∴ 10 + x = multiple of 3.


Hence, x = (multiple of 3) – 10


Here, possible values for multiples of 3 are 12, 15 and 18


Hence, x = 2, 5 or 8


Hence, possible numbers are 723, 753 and 783



Question 11.

Find all possible values of y for which the number 53y1 is divisible by 3. Also, find each such number.


Answer:

Here, our given number is 53y1, which is divisible by 3.


We know that if number is divisible by 3 only when sum of digits of given number is divisible by 3.


Hence, if 53y1 is divisible by 3, then,


5 + 3 + y + 1 = 0, 3, 6, 9,12,…


∴ 9 + y = multiple of 3.


Hence, y = (multiple of 3) – 9


Here, possible values for multiples of 3 are 9, 12, 15 and 18


Hence, y = 0, 3, 6 and 9


Hence, possible numbers are 5301, 5331, 5361 and 5391.


(In given answer, 5301 is not considered)



Question 12.

Find the value of x for which the number x806 is divisible by 9. Also, find the number.


Answer:

Here, our given number is x806, which is divisible by 9.


We know that if number is divisible by 9 only when sum of digits of given number is divisible by 9.


Hence, if x806 is divisible by 9, then,


X + 8 + 0 + 6 = 0, 9, 18,…


∴ 14 + x = multiple of 9.


Hence, x = (multiple of 9) – 14


Here, possible values for multiples of 9 is 18


Hence, x = 4


∴ Possible number is 4806.



Question 13.

Find the value of z for which the number 471z8 is divisible by 9. Also, find the number.


Answer:

Here, our given number is 471z8, which is divisible by 9.


We know that if number is divisible by 9 only when sum of digits of given number is divisible by 9.


Hence, if 471z8 is divisible by 9, then,


4 + 7 + 1 + z + 8 = 0, 9, 18,…


∴ 20 + z = multiple of 9.


Hence, z = (multiple of 9) – 20


Here, possible values for multiples of 9 is 27


Hence, z = 7


∴ Possible number is 47178



Question 14.

Give five examples of numbers, each one of which is divisible by 3 but not divisible by 9.


Answer:

Examples of numbers, such that each one of which is divisible by 3 but not divisible by 9, are 21, 24, 30, 33 and 39.



Question 15.

Give five examples of numbers, each one of which is divisible by 4 but not divisible by 8.


Answer:

Examples of numbers, such that each one of which is divisible by 4 but not divisible by 8, are 28, 36, 44, 52 and 60




Exercise 5c
Question 1.

Replace A, B, C by suitable numerals.



Answer:

Here, in units place,


A + 7 = 3


Ie, A = -4, which is not possible.


Hence A is greater than 10,where, 1 carry is given to tens place.


∴A + 7 = 13


∴ A = 6


Now in tens place,


5 + 8 + 1 = B …as 1 is carried


∴ B = 14


Here, 1 will be carried to hundreds place.


Hence, B = 4 and C = 1



Question 2.

Replace A, B, C by suitable numerals.



Answer:

Here, in units place,


6 + A = 3


Ie. A = - 3, which is not possible.


Hence A is greater than 10,where, 1 carry is given to tens place.


∴ A + 6 = 13


∴A = 7


Now, in tens place,


B + 9 + 1 = 7 … as 1 is carried


∴ B = -3, which is not possible.


Hence B is greater than 10,where, 1 carry is given to hundreds place.


∴ B + 9 + 1 = 17


∴ B = 7


Now, in hundreds place,


C + 6 + 1 = 1 … as 1 is carried


Ie. C = -6, which is not possible.


Hence C is greater than 10, where, 1 carry is given to thousands place.


∴ C + 6 + 1 = 11


∴ C = 4



Question 3.

Replace A, B by suitable numerals.



Answer:

Here, in units place,


A + A +A = A


Ie. 3A = A, ie. 3 = 1 which is wrong.


Hence A is greater than 10, where, 1 carry is given to tens place.


∴ A + A + A = A + 10


∴ 3A = A + 10


∴ 2A = 10


∴ A = 5


Now, in tens place,


B = 1 … as 1 is carried



Question 4.

Replace A, B by suitable numerals.



Answer:

Here, in tens place,


6 – 3 = 3


Which implies that maximum value of A is 3


Ie. A ≤ 3


Now in units place,


A - B = 7


This states that borrowing is involved.


∴ in tens place,


6 – A – 1 = 3


∴ A = 2


Now in units place,


A + 10 – B = 7 …as borrowing is involved


∴ 12 – B = 7


∴ B = 5



Question 5.

Replace A, B, C by suitable numerals.



Answer:

Now in units place,


5 - A = 9


This states that borrowing is involved.


Ie. 10 + 5 - A = 9


∴ A = 6


Now in tens place, as 1 is borrowed from hundreds place and also lent,


B – 5 +10 - 1 = 8


B = 4


Now in hundred place, as 1 is lent,


C – 2 – 1 = 2


C = 5



Question 6.

Replace A, B, C by suitable numbers:



Answer:

Here, (B × 3) = B


Here, B can be either 0 or 5, which satisfies above condition.


If B is 5, then 1 will be carried,


then, A×3+1 = A will not be possible for any number


∴ B = 0


Also, A×3=A is possible for either 0 or 5.


If we take A=0, then all number will become 0, which is not possible


.∴ A= 5


So, 1 will be carried.


∴ C = 1



Question 7.

Replace A, B, C by suitable numbers:



Answer:

Here, we can observe that B × A = B


i.e. A = 1


Here,


First digit = B+1
Thus, 1 will be carried from 1+B2 and becomes (B+1) (B2 -9) B.
∴ C = B2 -1
Now, all B, B+1 and B2 -9 are one digit number.
This condition is satisfied for B=3 or B=4.
For B< 3, B2 -9 will be negative.
For B>3, B2 -9 will become a two digit number.
For B=3, C = 32 - 9 = 9-9 = 0
For B = 4, C = 42 -9 = 16-9 = 7


Hence, A=1, B=3, C = 0 or A=1, B=4, C = 7



Question 8.

Replace A, B, C by suitable numerals.



Answer:

Here,


(A−4) = 3


∴ A = 7


Now, C ×6=36


∴ C = 6


Clearly, B = 6
∴ A=7, B=C=6



Question 9.

Find two numbers whose product is a 1-digit number and the sum is a 2-digit number.


Answer:

Clearly, for given condition which is, two numbers whose product is a 1-digit number and the sum is a 2-digit number, 1 and 9 satisfy.


Here,


1×9=9 and 1+9=10


Hence, 1 and 9 are required numbers



Question 10.

Find three whole numbers whose product and sum are equal.


Answer:

Clearly, for given condition which is, three whole numbers whose product and sum are equal, 1,2 and 3 satisfy.


Here, 1 + 2 +3 = 1 × 2 × 3 = 6



Question 11.

Complete the magic square given below, so that the sum of the numbers in each row or in each column or along each diagonal is 15.





Answer:

Solving for diagonal,


6 + 5 + x = 15


∴ x = 4



Solving for first row,


6 + 1 + x = 15


∴ x = 8



Solving for last column,


8 + x + 4 = 15


∴ x = 3



Solving for second row,


x + 5 + 3 = 15


∴ x = 7



Solving for first column,


6 + 7 + x = 15


∴ x = 2



Solving second column,


1 + 5 + x = 15


∴ x = 9





Question 12.

Fill in the numbers from 1 to 6 without repetition, so that each side of the triangle adds up to 12.



Answer:

Place the largest numbers, i.e., 4, 5 and 6, at the three corners of the triangle.
Now, 4 + 5 = 9, 4 + 6 = 10 and 5 + 6 = 11
Therefore, by placing 3 between 4 and 5, 2 between 4 and 6, and 1 between 5 and 6 we get the desired magic triangle.




Question 13.

Fibonacci numbers Take 10 numbers as shown below:

a, b, (a + b), (a + 2b), (2a + 3b), (3a + 5b), (5a + 8b), (8a + 13b), (13a + 21b), and (21a + 34 b). Sum of all these numbers =11(5a + 8b) = 1 1 x 7th number.

Taking a = 8, b =13; write 10 Fibonacci numbers and verify that sum of all these numbers =11 x 7th number.


Answer:

Here a = 8 and b = 13


The numbers in the Fibonacci sequence are arranged in the following manner:
1st, 2nd, (1st+2nd), (2nd+3th), (3th+4th), (4th+5th), (5th+6th), (6th+7th), (7th+8th), (8th+9th), (9th+10th)
The numbers are 8, 13, 21, 34, 55, 89, 144, 233, 377 and 610.
Sum of the numbers = ​8+13+21+34+55+89+144+233+377+610
=1584
11×7th number = 11×144=1584



Question 14.

Complete the magic square:


Answer:

Here, sum of last column = 0 + 11 + 7 + 12


Solving for last row,


x + 2 + 1 + 12 = 30


∴ x = 15



Solving for first column,


x + 8 + 4 + 15 = 30


∴ x = 3



Solving for first row,


3 + 14 + x + 0 =30


∴ x = 13



Solving for second row,


8 + x + 6 + 11 = 30


∴ x = 5



Solving for second column,


14 + 5 + x + 2 = 30


∴ x = 9



Solving for third column,


13 + 6 + x + 1 = 30


∴ x = 10





Exercise 5d
Question 1.

If 5x6 is exactly divisible by 3, then the least value of x is
A. 0

B. 1

C. 2

D. 3


Answer:

We know that if sum of digits of a number is divisible by 3, then the number is divisible by 3.


Here,


5 + x + 6 = multiple of 3


∴ 11 + x = 0, 3, 6, 9, ….


Hence,


11 + x = 12


∴ x= 1


the least value of x is 1


Question 2.

If 64y8 is exactly divisible by 3, then the least value of y is
A. 0

B. 1

C. 2

D. 3


Answer:

We know that if sum of digits of a number is divisible by 3, then the number is divisible by 3.


Here,


6 + 4 + y + 8 = multiple of 3


18 + y = 0, 3, 6, 9, ….


Hence,


18 + y = 18


∴ y = 0


the least value of y is 0.


Question 3.

If 7x8 is exactly divisible by 9, then the least value of x is
A. 0

B. 2

C. 3

D. 5


Answer:

We know that if sum of digits of a number is divisible by 9, then the number is divisible by 9.


Here,


7 + x + 8 = multiple of 9


15 + x = 0, 9, 18, …


Hence,


15 + x = 18


∴ x = 3


∴the least value of y is 3


Question 4.

If 37y4 is exactly divisible by 9, then the least value of y is
A. 2

B. 3

C. 1

D. 4


Answer:

We know that if sum of digits of a number is divisible by 9, then the number is divisible by 9.


Here,


3 + 7 + y + 4 = multiple of 9


y + 14 = 0, 9, 18, …


Hence,


y = 4


∴the least value of y is 3


Question 5.

If 4xy7 is exactly divisible by 3, then the least value of (x + y) is
A. 1

B. 4

C. 5

D. 7


Answer:

We know that if sum of digits of a number is divisible by 3, then the number is divisible by 3.


Here,


4 + x + y + 7 = multiple of 3


x + y + 11 = 0, 3, 6, 9, ….


Hence,


x + y + 11 = 12


∴ x + y = 1


∴ the least value of (x + y) is 1


Question 6.

If x7y5 is exactly divisible by 3, then the least value of (x + y) is
A. 6

B. 0

C. 4

D. 3


Answer:

We know that if sum of digits of a number is divisible by 3, then the number is divisible by 3.


Here,


x + 7 + y + 5 = multiple of 3


x + y + 12 = 0, 3, 6, 9, ….


Hence,


x + y + 12 = 12


∴ x + y = 0


But x + y cannot be 0 because then x and y both will have to be 0.
Since x is the first digit, it cannot be 0.
∴ x + y + 12 = 15


x + y = 3


∴ the least value of (x + y) is 3


Question 7.

If x4y5z is exactly divisible by 9, then the least value of (x + y + z) is
A. 3

B. 6

C. 9

D. 0


Answer:

We know that if sum of digits of a number is divisible by 9, then the number is divisible by 9.


Here,


x + 4 + y + 5 + z = multiple of 9


x + y + z + 9 = 0, 9, 18, ….


Hence,


x + y + z + 18 = 18


But x + y + z cannot be 0 because then x, y and z will have to be 0.
Since x is the first digit, it cannot be 0.
∴ x + y + z + 18 = 27


x + y + z = 9


∴ the least value of (x + y) is 9


Question 8.

If 1A2B5 is exactly divisible by 9, then the least value of (A + B) is
A. 0

B. 1

C. 2

D. 10


Answer:

We know that if sum of digits of a number is divisible by 9, then the number is divisible by 9.


Here,


1 + A + 2 + B + 5 = multiple of 9


A + B + 8 = 0, 9, 18, ….


Hence,


A + B + 8 = 9


A + B =1


Question 9.

If the 4-digit number x27y is exactly divisible by 9, then the least value of (x + y) is
A. 0

B. 3

C. 6

D. 9


Answer:

We know that if sum of digits of a number is divisible by 9, then the number is divisible by 9.


Here,


x + 2 + 7 + y = multiple of 9


x + y + 9= 0, 9, 18, …


Hence,


x + y + 9 = 9


∴ x + y =0


But x + y cannot be 0 because then x and y both will have to be 0.
Since x is the first digit, it cannot be 0.
∴ x + y + 9 = 18


x + y = 9


∴ the least value of (x + y) is 9



Cce Test Paper-5
Question 1.

Find all possible values of x for which the 4-digit number 320x is divisible by 3. Also, find the numbers.


Answer:

We know that if sum of digits of a number is divisible by 3, then the number is divisible by 3.


Here,


3 + 2 + 0 + x = multiple of 3


5 + x = 0, 3, 6, 9, 12, 15, ….


Hence,


1) 5 + x = 6


∴ x = 1


2) 5 + x = 9


∴ x = 4


3) 5 + x = 12


∴ x = 7


Hence, possible number 3201, 3204 and 3207.



Question 2.

Find all possible values of y for which the 4-digit number 64y3 is divisible by 9. Also, find the numbers.


Answer:

We know that if sum of digits of a number is divisible by 9, then the number is divisible by 9.


Here,


6 + 4 + y + 3 = multiple of 9


y + 13= 0, 9, 18, …


Hence,


y + 13 = 18


∴ y = 5


The number will be 6453.



Question 3.

The sum of the digits of a 2-digit number is 6. The number obtained by interchanging its digits is 18 more than the original number. Find the original number.


Answer:

Let the two numbers of the two-digit number be 'a' and 'b'.
a + b = 6 ... (1)
The number can be written as ( 10a + b ).
After interchanging the digits, the number becomes ( 10b + a ).
( 10a + b ) + 18 = ( 10b + a ) 9a − 9b = −18
a − b = −2 ... (2)
Adding equations (1) and (2):
2a = 4


∴ a = 2
Using a = 2 in equation (1),
b = 6 – a = 6 – 2 = 4
Therefore, the original number is 24.



Question 4.

Which of the following numbers are divisible by 9?

(i) 524618

(ii) 7345845

(iii) 8987148


Answer:

We know that if sum of digits of a number is divisible by 9, then the number is divisible by 9.


Here, sum of digits is 26, which is not divisible by 9.


Hence, 524618 is not divisible by 9.


(ii) 7345845


We know that if sum of digits of a number is divisible by 9, then the number is divisible by 9.


Here, sum of digits is 36, which is divisible by 9.


Hence, 7345845 is divisible by 9.


(iii) 8987148


We know that if sum of digits of a number is divisible by 9, then the number is divisible by 9.


Here, sum of digits is 45, which is divisible by 9.


Hence, 8987148 is divisible by 9.



Question 5.

Replace A, B, C by suitable numerals:



Answer:

Here, in units place,


A – 8 = 3
​This implies that 1 is borrowed.
11 – 8 = 3


∴ A = 1
Now in tens place,
Then, 7−B=9
1 is borrowed from 7.
∴ 16−B=9


∴ B = 7
Now in hundreds place,


5−C=2
But 1 has been borrowed from 5.
∴ 4 - C = 2
∴ C=2
∴ A=1, B=7 and C=2



Question 6.

Replace A, B, C by suitable numerals:



Answer:

Here,


(A−6) = 6


∴ A = 12


A can not be two digit number.


Hence, 1 will be borrowed.


Now, C ×7=63


∴ C = 9


Clearly, B = 3
∴ A=7, B=C=6



Question 7.

Find the values of A, B, C when



Answer:

A × B = B


∴ A = 1


∴ C = 1 + B2


Also 1 + B2 is a single digit.


∴ B = 2


C = ( 1 + B2 ) = ( 1 + 4 ) = 5


∴ A = 1, B = 2 and C = 5



Question 8.

If 7 × 8 is exactly divisible by 3, then the least value of x is
A. 3

B. 0

C. 6

D. 9


Answer:

We know that if sum of digits of a number is divisible by 3, then the number is divisible by 3.


Here,


7 + x + 8 = multiple of 3


x + 15 = 0, 3, 6, 9, ….


Hence,


x + 15 = 15


x = 0


∴ the least value of x is 0


Question 9.

If 6x5 is exactly divisible by 9, then the least value of x is
A. 1

B. 4

C. 7

D. 0


Answer:

We know that if sum of digits of a number is divisible by 9, then the number is divisible by 9.


Here,


6 + x + 5 = multiple of 9


x + 11 = 0, 9, 18, ….


Hence,


x + 11 = 18


∴ x = 7


∴ the least value of x is 7


Question 10.

If x48y is exactly divisible by 9, then the least value of (x + y) is
A. 4

B. 0

C. 6

D. 7


Answer:

We know that if sum of digits of a number is divisible by 9, then the number is divisible by 9.


Here,


x + 4 + 8 + y = multiple of 9


x + y + 12 = 0, 9, 18, ….


Hence,


x + 11 = 18


∴ x = 7


∴ the least value of x is 7


Question 11.

If 486*7 is divisible by 9, then the least value of * is
A. 0

B. 1

C. 3

D. 2


Answer:

We know that if sum of digits of a number is divisible by 9, then the number is divisible by 9.


Here, for the given number to be divisible by 9


4 + 8 + 6 + * + 7 = multiple of 9


* + 25 = 0, 9, 18, ….(multiple of 9)


Hence,


* + 25 = 27 [ lowest multiple then]
* = 2


∴ x = 2


∴ the least value of x is 2